If $sum_{n=1}^{infty}a_n$ converges then $sum_{n=1}^{infty}a_n+a_{n+1}+a_{n+2}$ converges












1












$begingroup$


I have the following problem:



Let $a_n$, $ninmathbb{N}$ be sequence
and let $b_n=a_n+a_{n+1}+a_{n+2}$



Prove that if $sum_{n=1}^{infty}a_n$ converges then $sum_{n=1}^{infty}b_n$ converges



My attempt:



Let $sum_{n=1}^{infty}a_n=a$



$sum_{n=1}^{infty}b_n=sum_{n=1}^{infty}a_n+a_{n+1}+a_{n+2}$=



$sum_{n=1}^{infty}a_n+sum_{n=1}^{infty}a_{n+1}+sum_{n=1}^{infty}a_{n+2}$=



$sum_{n=1}^{infty}a_n$+$sum_{n=1}^{infty}a_n-a_1$+$sum_{n=1}^{infty}a_n-a_1-a_2$=



$sum_{n=1}^{infty}3a_n-2a_1-a_2$



From the linearity of series we know that $sum_{n=1}^{infty}3a_n=3a$



And the series convergence isn't affected by a change in finite number of elements of the sum



So $sum_{n=1}^{infty}b_n$ converges



Is this any good? Is it sufficient?










share|cite|improve this question









$endgroup$








  • 2




    $begingroup$
    Your proof is correct. Here's a more general proof: if $sum a_n, sum b_n$ converge, then $sum a_n + b_n$ converges and equals $sum a_n + sum b_n$. Use this and the fact that $sumlimits_{n=1}^infty a_{n+k}$ converges for every k if $sum a_n$ exists. You basically used these statements within your proof.
    $endgroup$
    – James Yang
    Jan 22 at 20:32












  • $begingroup$
    Yes your proof is correct
    $endgroup$
    – Mike
    Jan 22 at 20:35










  • $begingroup$
    Please put in parentheses.
    $endgroup$
    – zhw.
    Jan 22 at 21:04
















1












$begingroup$


I have the following problem:



Let $a_n$, $ninmathbb{N}$ be sequence
and let $b_n=a_n+a_{n+1}+a_{n+2}$



Prove that if $sum_{n=1}^{infty}a_n$ converges then $sum_{n=1}^{infty}b_n$ converges



My attempt:



Let $sum_{n=1}^{infty}a_n=a$



$sum_{n=1}^{infty}b_n=sum_{n=1}^{infty}a_n+a_{n+1}+a_{n+2}$=



$sum_{n=1}^{infty}a_n+sum_{n=1}^{infty}a_{n+1}+sum_{n=1}^{infty}a_{n+2}$=



$sum_{n=1}^{infty}a_n$+$sum_{n=1}^{infty}a_n-a_1$+$sum_{n=1}^{infty}a_n-a_1-a_2$=



$sum_{n=1}^{infty}3a_n-2a_1-a_2$



From the linearity of series we know that $sum_{n=1}^{infty}3a_n=3a$



And the series convergence isn't affected by a change in finite number of elements of the sum



So $sum_{n=1}^{infty}b_n$ converges



Is this any good? Is it sufficient?










share|cite|improve this question









$endgroup$








  • 2




    $begingroup$
    Your proof is correct. Here's a more general proof: if $sum a_n, sum b_n$ converge, then $sum a_n + b_n$ converges and equals $sum a_n + sum b_n$. Use this and the fact that $sumlimits_{n=1}^infty a_{n+k}$ converges for every k if $sum a_n$ exists. You basically used these statements within your proof.
    $endgroup$
    – James Yang
    Jan 22 at 20:32












  • $begingroup$
    Yes your proof is correct
    $endgroup$
    – Mike
    Jan 22 at 20:35










  • $begingroup$
    Please put in parentheses.
    $endgroup$
    – zhw.
    Jan 22 at 21:04














1












1








1





$begingroup$


I have the following problem:



Let $a_n$, $ninmathbb{N}$ be sequence
and let $b_n=a_n+a_{n+1}+a_{n+2}$



Prove that if $sum_{n=1}^{infty}a_n$ converges then $sum_{n=1}^{infty}b_n$ converges



My attempt:



Let $sum_{n=1}^{infty}a_n=a$



$sum_{n=1}^{infty}b_n=sum_{n=1}^{infty}a_n+a_{n+1}+a_{n+2}$=



$sum_{n=1}^{infty}a_n+sum_{n=1}^{infty}a_{n+1}+sum_{n=1}^{infty}a_{n+2}$=



$sum_{n=1}^{infty}a_n$+$sum_{n=1}^{infty}a_n-a_1$+$sum_{n=1}^{infty}a_n-a_1-a_2$=



$sum_{n=1}^{infty}3a_n-2a_1-a_2$



From the linearity of series we know that $sum_{n=1}^{infty}3a_n=3a$



And the series convergence isn't affected by a change in finite number of elements of the sum



So $sum_{n=1}^{infty}b_n$ converges



Is this any good? Is it sufficient?










share|cite|improve this question









$endgroup$




I have the following problem:



Let $a_n$, $ninmathbb{N}$ be sequence
and let $b_n=a_n+a_{n+1}+a_{n+2}$



Prove that if $sum_{n=1}^{infty}a_n$ converges then $sum_{n=1}^{infty}b_n$ converges



My attempt:



Let $sum_{n=1}^{infty}a_n=a$



$sum_{n=1}^{infty}b_n=sum_{n=1}^{infty}a_n+a_{n+1}+a_{n+2}$=



$sum_{n=1}^{infty}a_n+sum_{n=1}^{infty}a_{n+1}+sum_{n=1}^{infty}a_{n+2}$=



$sum_{n=1}^{infty}a_n$+$sum_{n=1}^{infty}a_n-a_1$+$sum_{n=1}^{infty}a_n-a_1-a_2$=



$sum_{n=1}^{infty}3a_n-2a_1-a_2$



From the linearity of series we know that $sum_{n=1}^{infty}3a_n=3a$



And the series convergence isn't affected by a change in finite number of elements of the sum



So $sum_{n=1}^{infty}b_n$ converges



Is this any good? Is it sufficient?







real-analysis sequences-and-series convergence






share|cite|improve this question













share|cite|improve this question











share|cite|improve this question




share|cite|improve this question










asked Jan 22 at 20:23









ChangaChanga

234




234








  • 2




    $begingroup$
    Your proof is correct. Here's a more general proof: if $sum a_n, sum b_n$ converge, then $sum a_n + b_n$ converges and equals $sum a_n + sum b_n$. Use this and the fact that $sumlimits_{n=1}^infty a_{n+k}$ converges for every k if $sum a_n$ exists. You basically used these statements within your proof.
    $endgroup$
    – James Yang
    Jan 22 at 20:32












  • $begingroup$
    Yes your proof is correct
    $endgroup$
    – Mike
    Jan 22 at 20:35










  • $begingroup$
    Please put in parentheses.
    $endgroup$
    – zhw.
    Jan 22 at 21:04














  • 2




    $begingroup$
    Your proof is correct. Here's a more general proof: if $sum a_n, sum b_n$ converge, then $sum a_n + b_n$ converges and equals $sum a_n + sum b_n$. Use this and the fact that $sumlimits_{n=1}^infty a_{n+k}$ converges for every k if $sum a_n$ exists. You basically used these statements within your proof.
    $endgroup$
    – James Yang
    Jan 22 at 20:32












  • $begingroup$
    Yes your proof is correct
    $endgroup$
    – Mike
    Jan 22 at 20:35










  • $begingroup$
    Please put in parentheses.
    $endgroup$
    – zhw.
    Jan 22 at 21:04








2




2




$begingroup$
Your proof is correct. Here's a more general proof: if $sum a_n, sum b_n$ converge, then $sum a_n + b_n$ converges and equals $sum a_n + sum b_n$. Use this and the fact that $sumlimits_{n=1}^infty a_{n+k}$ converges for every k if $sum a_n$ exists. You basically used these statements within your proof.
$endgroup$
– James Yang
Jan 22 at 20:32






$begingroup$
Your proof is correct. Here's a more general proof: if $sum a_n, sum b_n$ converge, then $sum a_n + b_n$ converges and equals $sum a_n + sum b_n$. Use this and the fact that $sumlimits_{n=1}^infty a_{n+k}$ converges for every k if $sum a_n$ exists. You basically used these statements within your proof.
$endgroup$
– James Yang
Jan 22 at 20:32














$begingroup$
Yes your proof is correct
$endgroup$
– Mike
Jan 22 at 20:35




$begingroup$
Yes your proof is correct
$endgroup$
– Mike
Jan 22 at 20:35












$begingroup$
Please put in parentheses.
$endgroup$
– zhw.
Jan 22 at 21:04




$begingroup$
Please put in parentheses.
$endgroup$
– zhw.
Jan 22 at 21:04










0






active

oldest

votes











Your Answer





StackExchange.ifUsing("editor", function () {
return StackExchange.using("mathjaxEditing", function () {
StackExchange.MarkdownEditor.creationCallbacks.add(function (editor, postfix) {
StackExchange.mathjaxEditing.prepareWmdForMathJax(editor, postfix, [["$", "$"], ["\\(","\\)"]]);
});
});
}, "mathjax-editing");

StackExchange.ready(function() {
var channelOptions = {
tags: "".split(" "),
id: "69"
};
initTagRenderer("".split(" "), "".split(" "), channelOptions);

StackExchange.using("externalEditor", function() {
// Have to fire editor after snippets, if snippets enabled
if (StackExchange.settings.snippets.snippetsEnabled) {
StackExchange.using("snippets", function() {
createEditor();
});
}
else {
createEditor();
}
});

function createEditor() {
StackExchange.prepareEditor({
heartbeatType: 'answer',
autoActivateHeartbeat: false,
convertImagesToLinks: true,
noModals: true,
showLowRepImageUploadWarning: true,
reputationToPostImages: 10,
bindNavPrevention: true,
postfix: "",
imageUploader: {
brandingHtml: "Powered by u003ca class="icon-imgur-white" href="https://imgur.com/"u003eu003c/au003e",
contentPolicyHtml: "User contributions licensed under u003ca href="https://creativecommons.org/licenses/by-sa/3.0/"u003ecc by-sa 3.0 with attribution requiredu003c/au003e u003ca href="https://stackoverflow.com/legal/content-policy"u003e(content policy)u003c/au003e",
allowUrls: true
},
noCode: true, onDemand: true,
discardSelector: ".discard-answer"
,immediatelyShowMarkdownHelp:true
});


}
});














draft saved

draft discarded


















StackExchange.ready(
function () {
StackExchange.openid.initPostLogin('.new-post-login', 'https%3a%2f%2fmath.stackexchange.com%2fquestions%2f3083654%2fif-sum-n-1-inftya-n-converges-then-sum-n-1-inftya-na-n1a-n%23new-answer', 'question_page');
}
);

Post as a guest















Required, but never shown

























0






active

oldest

votes








0






active

oldest

votes









active

oldest

votes






active

oldest

votes
















draft saved

draft discarded




















































Thanks for contributing an answer to Mathematics Stack Exchange!


  • Please be sure to answer the question. Provide details and share your research!

But avoid



  • Asking for help, clarification, or responding to other answers.

  • Making statements based on opinion; back them up with references or personal experience.


Use MathJax to format equations. MathJax reference.


To learn more, see our tips on writing great answers.




draft saved


draft discarded














StackExchange.ready(
function () {
StackExchange.openid.initPostLogin('.new-post-login', 'https%3a%2f%2fmath.stackexchange.com%2fquestions%2f3083654%2fif-sum-n-1-inftya-n-converges-then-sum-n-1-inftya-na-n1a-n%23new-answer', 'question_page');
}
);

Post as a guest















Required, but never shown





















































Required, but never shown














Required, but never shown












Required, but never shown







Required, but never shown

































Required, but never shown














Required, but never shown












Required, but never shown







Required, but never shown







Popular posts from this blog

Can a sorcerer learn a 5th-level spell early by creating spell slots using the Font of Magic feature?

Does disintegrating a polymorphed enemy still kill it after the 2018 errata?

A Topological Invariant for $pi_3(U(n))$